2
$\begingroup$

Let $B$ denote the unit Euclidean ball centered at $0 \in \mathbb{R}^n$. Given a set $K \subset \mathbb{R}^n$ let us denote by $P_K(x)$ the maximal number of points $y_i \in K \cap (x + B)$ such that the Euclidean distance between $y_i, y_j$ is strictly greater than $1/2$. In other words it measures how many well-separated points we can put locally around $x$ in $K$.

Question: Suppose $K$ is sign-symmetric ($K = -K$), convex, and compact. Is it true that $P_K$ is maximized with $x = 0$?

Intuitively, this should be the case, since (assuming additionally, $K$ has nonempty interior) $K$ has maximal volume near the origin. Indeed, we know that $\mathrm{vol}_n(K \cap (x + B))$ is maximized at $x = 0$. So, intuitively the origin is the most "massive" and therefore should lead to the largest packing numbers.

One easy observation is that for $x \in K$, we can write $y \in K \cap (x + B)$ as $x + 2m$ where $m = (y-x)/2$. On the other hand, clearly $2m \in 2K \cap B$ under our assumptions. In other words $K \cap (x + B) \subset x + 2K \cap B$. This gives the slightly weaker inequality $$ \sup_{x \in K} P_K(x) \leq P_{2K}(0). $$

$\endgroup$

1 Answer 1

3
$\begingroup$

This does not look true, at least if we replace $1/2$ by a different constant.

E.g., assume that we replace it with the number a bit less than the side $\ell$ of a regular tetrahedron inscribed into $B$, in $\mathbb R^3$. Take such a tetrahedron $T$ inscribed into $x+B$ with vertices $a$, $b$, $c$, $e$ such that the altitude from $a$ passes through $0$, $a$ is the most distant from $0$, and $x$ is sufficiently far away from $0$. Let $K$ be the convex hull of $T\cup(-T)$. Then $P_K(x)=4$. On the other hand, the neighborhood of $0$ in $K$ looks almost as a hexagonal prism with the main diagonal of the base equal to $\ell$. If we had $P_K(0)=4$, that would mean that this prism contains an almost regular tetrahedron with side length $\ell$, which does not seem true.

I suspect that a similar example should exist for $1/2$ also; but it may be much harder to justify it.

$\endgroup$
2
  • $\begingroup$ Perhaps I misunderstood, in my head the object you described sounds like two tetrahedrons (one upside down) connected via a triangular prism. Is that correct? Or am I misunderstanding your $K$? $\endgroup$ Commented Sep 11 at 7:33
  • $\begingroup$ That is not a prism but an antiprism, as the tetrahedra are symmetric to each other in a point. $\endgroup$ Commented Sep 11 at 8:04

You must log in to answer this question.

Start asking to get answers

Find the answer to your question by asking.

Ask question

Explore related questions

See similar questions with these tags.